Applications of quaternions

Quaternions were discovered by Sir William Rowan Hamilton in a flash of inspiration as he crossed Brougham Bridge, inscribing the following relations into one of the stones:

i² = j² = k² = ijk = −1

Specifically, a quaternion is a number of the form qabicjdk, where a, b, c, and d are real and the elements ij and k satisfy the above relations.

Note that multiplication of quaternions is non-commutative, but the other field axioms hold as expected. In particular, addition forms an Abelian group, multiplication is associative and distributes over addition, and we have a multiplicative identity, 1. Moreover, the real line occurs as a subspace of the quaternions; this is precisely the centre of the multiplicative group.

The notions of ‘absolute value’ and ‘complex conjugation’ generalise to quaternions in the obvious way. Specifically, the multiplicative inverse of q is given by:

q^-1 = q*/|q|² = (a − bi − cj − dk)/(a² + b² + c² + d²)

Also, note that if q is a non-real quaternion, the R-linear subspace spanned by 1 and q is isomorphic to the field of complex numbers.

Rotations in \mathbb{R}^3 and \mathbb{R}^4

In two dimensions, a general rotation can be expressed in the form z → az, where a is a complex number of unit norm. One may naïvely assume that the same result holds for quaternions. However, this is untrue: we require a pair of unit quaternions, (a, b), to represent a general rotation q → aqb*. Moreover, (ab) and (-a, –b) represent the same rotation, so S³ × S³ is isomorphic to Spin(4), the double-cover of SO(4).

The diagonal subgroup {(a, a) : a in S³} has the property of fixing all points on the real line. Consequently, the three-space spanned by {i, j, k} is preserved by this subgroup, so we can represent three-dimensional rotations by unit quaternions. Indeed, S³ is isomorphic to Spin(3), the double-cover of SO(3).

Question 3 of NST1 2011

The following question appeared on a selection test for the British IMO team:

NST2011

 

See if you can solve it (solutions are welcome in the comments at the foot of this page).

 

This entry was posted in Uncategorized. Bookmark the permalink.

12 Responses to Applications of quaternions

  1. If “rational coefficients” allows complex ones with rational components, then n = 3; f1(x) = x, f2(x) = 4, f3(x) = 3i. Otherwise, it looks like n is at most 5: f1(x) = x, f2(x) = 2, f3(x) = f4(x) = f5(x) = 1.

    Note that the x^2 seems to have to come from a bare x as one of the polynomials; if f_k(x) was ax + b, it would contribute a^2x^2 + 2abx + b^2 and we’d have to get rid of the 2abx somehow. The only way is with more polynomials with the same structure, ending up with sum(a_k^2) = 1, sum(a_kb_k) = 0, and (if we want no other fs) sum(b_k^2) = 7. The latter requires an integer solution to sum(x_k^2) = 7y^2, though (where y is a common multiple of the denominators of the b_k, and the x_k are the b_k times y are integers). That’s impossible, for n < 4 though. If y is written as s*2^r where s is odd, y^2 = s^2*4^r, and s^2 is congruent to 1 mod 8, so 7s^2 is congruent to 7 mod 8 and 7y^2 runs afoul of Legendre's Three Square Theorem, excluding n = 3; 7 being not a square excludes n = 1 and 7 being congruent to 3 mod 4 excludes n = 2.

    For n = 4 it seems like it *should* be possible. We need a, b, c, d whose squares sum to 1 and e, f, g, h whose squares sum to 7, with the property that ae + bf + cg + dh = 0. This turns out to be equivalent to finding a, b, c, d whose squares sum to a perfect square and e, f, g, h whose squares sum to 7 times a perfect square, in integers, with the same constraint ae + bf + cg + dh = 0.

    Conjugacy considerations (mainly mod 4) indicate that for a primitive solution (a, b, c, d lack a common factor and e, f, g, h lack a common factor), a, b, c, d are either all odd or three even, one odd, and e, f, g, h are either all odd or one even, three odd. This does not seem to preclude a solution existing. I have not been able to find a solution in small integers to this, however.

    So, n may be 4 and is definitely either 4 or 5.

  2. I think I now have a proof that n must be 5. If n were 4, we could take the coefficients of x from our polynomials and make of them a unit quaternion w with rational coordinates. The coefficients of 1 would form a quaternion z with norm sqrt(7). The ae + bf + cg + dh = 0 criterion amounts to “wz is pure imaginary”. But wz also has norm sqrt(7), so we’d need to have the sum of three squares, the imaginary-component magnitudes of wz, adding to 7, which is impossible by Legendre’s Three Square Theorem.

    So n = 5, and x, 2, 1, 1, 1 is a minimal solution (though not unique; there’s also x, 3/2, 3/2, 3/2, 1/2; x, 9/5, 7/5, 6/5, 3/5; x, 2, 5/3, 1/3, 1/3; x, 5/3, 5/3, 1, 2/3; x, 13/5, 2/5, 1/5, 1/5; x, 5/2, 1/2, 1/2, 1/2; x, 2, 7/5, 1, 1/5; and, it seems likely, infinitely many more like this, as well as ones with more interesting degree-1 polynomials than just a bare number or x).

    • apgoucher says:

      Well done! I think that your proof may be even neater than my (similarly quaternionic, and probably equivalent) proof, which was:

      “Let (w, z) be a solution (using your notation). The necessary and sufficient conditions are that |w|^2 = 1, |z|^2 = 7, and w . z = 0 (in the Euclidean inner product). Hence we can premultiply w and z by w* to give another solution (ww*, zw*). But ww* is purely real, so zw* is purely imaginary. Contradiction.”

      However, my proof does provide a way to generate an infinite family (and possibly all) solutions — just `rotate’ your initial solution (w, z) by any pair of unit quaternions with rational coefficients. And we can enumerate all of *those* by the same `inversion’ technique for enumerating Pythagorean triples, but in R^4 instead of R^2..

    • First a clarification: ae + bf + cg + dh = 0 amounts to “wz is pure imaginary” since we can flip the signs of f, g, and h say, without altering the sums of squares, and get ae – bf – cg – dh. So given any solution you’d be able to derive from it a pair of quaternions w and z with the requisite property.

      Second, if this is generalized to x^2 + m for some positive integer m, it seems that n is 5 if m runs afoul of Legendre’s Three Square Theorem, but is never higher (since all positive integers are sums of four squares, one can always find an x, a, b, c, d solution for any positive integer m); if m divided by its highest internal power of four is not congruent to 7 mod 8, n is at most 4. n = 1 for only m = 0 and n = 2 if k is a perfect square — even if we have general ax + b and cx + d, to get (a^2 + c^2)x = x, a^2 and c^2 end up being p/r and q/r where p, q, r form a Pythagorean triplet, b:d = – c:a, so there is some s such that c = -bs and d = as, and thence b^2 + d^2 = r^2s^2 = m is a perfect square. If m is a non-square sum of two squares, then obviously n = 3, and the question that remains is if the set of m for which n = 3 is precisely that or is some larger subset of non-Legendre-affected m. Now, consider the quaternions again, with |z| = 1, |w|^2 = m, wz pure imaginary, and w’s and z’s fourth components equal to zero, so z = a + bi + cj and w = e + fi + gj. In that case, wz being pure imaginary may still have three nonzero components, as for example the expansion for zw includes a term bgk that need not be zero. So if m is non-Legendre-affected this and the underconstrained nature of the system otherwise (six unknowns in three equations!) strongly hints that there will be solutions with n = 3. So n = 4 *may* not occur for any m … though, I don’t know for sure.

      • apgoucher says:

        What makes it difficult is the lack of a three-dimensional normed division algebra. This is what Hamilton was seeking before he found the quaternions, I seem to recall.

        I conjecture that n = 4 in the case of m = 3, mainly because my attempts to find a solution for three polynomials failed.

      • apgoucher says:

        Sorry, what was I thinking? *Obviously* x^2 + 3 requires four polynomials; just substitute x = 2 and appeal to Legendre!

        • Yeah, that works. So n = 4 does occur, for (at least) the subset of m where m itself doesn’t run afoul of Legendre, but there exists some integer s with s^2 + m doing so. This is some subset of the m for which m is a sum of three squares, since if m is a sum of two squares, s^2 + m cannot run into Legendre and besides we have the n = 3 solution [x, square 1, square 2].

          Actually, it looks likely to be *all* the m that are sums of three squares but not of fewer that have n = 4. If every sum of three squares occurs as three of the four squares adding up to some Legendre-number, then that will be the case. Only if there is some a^2 + b^2 + c^2 with the property that for *all* integers d, a^2 + b^2 + c^2 + d^2 can be rewritten as e^2 + f^2 + g^2 (in integers!), might we have an instance where m is a three-square number but n = 3.

          Congruency considerations narrow down when that might happen. The residues mod 8 are 0, 1, and 4; the possible signatures of our triplet with respect to this (up to rearrangement) are [0 0 0], [0 0 1], [0 0 4], [0 1 1], [0 1 4], [0 4 4], [1 1 1], [1 1 4], [1 4 4], [4 4 4]. Of these:

          [0 0 0], [0 4 4] are divisible by 8 and adding a d^2 with residue 4 is likely to allow getting 4*(8r + 7).

          [0 0 4], [4 4 4] are odd multiples of 4 and adding a d^2 with residue 0 is likely to allow getting 4*(8r + 7).

          [1 1 1] is congruent to 3 mod 8; just add a d^2 that’s an odd multiple of 4 (such as, say, 4) and the result is odd and congruent to 7 mod 8. Adding a d^2 that’s odd would produce an odd multiple of 4, and is likely to further allow getting 4*(8r + 7).

          [1 1 4] is congruent to 6 mod 8; just add a d^2 that’s odd (such as, say, 1) and the result is odd and congruent to 7 mod 8.

          [0 0 1], [1 4 4] are congruent to 1 mod 8.

          [0 1 1] is congruent to 2 mod 8.

          [0 1 4] is congruent to 5 mod 8.

          So, the sums of three squares (but no fewer) that will have n = 4 are congruent to 0, 3, 4, and 6 mod 8; sums that *may* possibly have n = 3 (not ruled out yet anyway) are congruent to 1, 2, and 5 mod 8.

          Of course, sums of *two* squares are [0 0], [0 1], [0 4], [1 1], [1 4], [4 4], with residues 0, 1, 2, 4, and 5, which leads to the sneaking suspicion that the sums that have n = 3 (and thus have residue sum 1, 2, or 5) are probably identical with the ones that can be re-expressed as a sum of *two* squares…

          Thus, I conjecture now that for x^2 + m, n = (one more than the minimum number of squares needed to sum to m). Which has the corollary “there is always an optimal solution of the form [x, a, b, c, d] with possibly fewer constant polynomials; never does the optimal solution set contain *only* sets with a nontrivial ax + b polynomial, i.e. with neither a nor b being zero”.

          (Incidentally, the above actually implies a partial *proof* of that Legendre theorem. The sums of three squares give you every residue mod 8 except 7, proving the nonexistence of three square sum forms for s = 0 in 4^s(8r + 7); when s > 0 there cannot be any 1s in the squares’ residues (as there would need to be four of them, out of a maximum of three!) so all of the squares divide by 4 and one can inductively divide all of the squared values by 2 and get a solution for 4^(s – 1)(8r + 7). The proof of existence for numbers *not* of this form proceeds, after removing powers of 4, by extending numbers congruent to 0, 3, 4, or 6 mod 8 to ones congruent to 7 mod 8 by adding a square, and then using the four square theorem; for the ones congruent to 1, 2, or 5, by finding the suspected *two* square solution and adding a 0^2 term.)

          • 77 is a sum of three squares (8^2 + 3^2 + 2^2), but not of 2 squares (they can’t both be 72, so one is either 49 or 64, but 77 – 49 = 28 and 77 – 64 = 13), and is congruent to 5 mod 8, so any extension to a sum of four squares is congruent to 1, 5, or 6 mod 8 — while Legendre-problem numbers are all congruent to 0 or 7 mod 8. So x^2 + 77 is still a candidate for having n = 3 with no [x, constant, constant] solution in the solution set…

  3. Ooookay, *something* screwy happened. That last one is missing a chunk of text between “be” and “72” which I definitely did not forget to include, but which disappeared somehow in transit, and is also missing its “reply” link… I *meant* to say that they can’t both be 36 or lower, as 77 is greater than 72, not that they can’t both be 72 (although they indeed can’t both be 72). And I *don’t* appreciate my comments being altered, post-“send”, by someone-or-something-other-than-me in ways that alter the meaning in a way that makes me look stupid and bad at math. I assume some dodgy automation is responsible, rather than the alteration being intentional on anyone’s part, but the effect is the same…

  4. I’ve got a bit more. 77 (and also 66, and a few other numbers) run afoul of a different conjugacy “law”. The residues mod 11 are 0, 1, 3, 4, 5, and 9 — no pair of which sum to 11, other than two 0s. Thus, a sum of two squares is only ever divisible by 11 if the individual squares were both divisible by 11. From that observation, and the observation that 121 is congruent to 1 mod 8, one can infer that a number of the form 121^r*4^s*(8t + u), with neither 4 nor 121 dividing 8t + u, is not expressible as a sum of fewer than three squares if either 8t + u is divisible by 11, u = 3, u = 6, or u = 7 (and if u = 7 it requires four squares).

    • apgoucher says:

      The necessary and sufficient condition for N to be a sum of two squares is that N = h m^2, where m is an arbitrary integer and h is not divisible by any primes of the form 4k + 3.

      This eliminates the need for excessive case-bashing.

Leave a Reply to apgoucherCancel reply